please simplify this:(5x^4)^3

Answers

Answer 1

[tex](5x^{4} )^{3} \\=5x^{4*3} \\=5x^{12}[/tex]

I used the law of exponents as indicated below:

[tex](x^{n} )^{m} \\=x^{n*m} \\=x^{nm}[/tex]


Related Questions

Label the points, lines, and planes to show AB and line n perpendicular to each other in plane R at point B, plane R
(
AB both perpendicular to line p and intersecting line m at point A.
intersecting plane S in line p, and
R

Answers

Answer:

sorry i don't know it to should have listened to your teacher

Step-by-step explanation:

Label the points,lines,and planes to show AB and line in perpendicular

to each other in plane R to point B, plane R

Describe the graph of y = -|x + 4| - 5

Answers

     Trigonometry is the study of angles and the angular relationships between planar and three-dimensional shapes. The cosecant, cosine, cotangent, secant, sine, and tangent are among the trigonometric functions (also known as the circle functions) that make up trigonometry.

What is y =-(x 5)+4's vertex? Y = | X is it linear?

Examples of algebra. Discover the vertex of absolute value. The vertex in this instance for y=|x5|+4 y = | x - 5 | + 4 is (5,4). Set the interior of the absolute value x5 x - 5 equal to 0 0 to determine the vertex's x x coordinate.

Examples of Algebra The degree of variable y in this instance is 1, which means that the degrees of the variables in the equation violate the definition of a linear equation, indicating that the equation is not a linear equation.

Pick a few x values and look for some ordered pairings to graph an absolute value function. Connect the points by plotting them on a coordinate plane. The graph has a V form, as you can see. (1) The graph's vertex is (0,0).

Obtain the equation y = ax2 + bx + c.

Determine -b / 2A. The vertex's x-coordinate is given here.

Simply enter the value of -b / 2a into the equation for x and solve for y to obtain the vertex's y-coordinate. The vertex's y-coordinate is given here.

To Learn more About y = -|x + 4| - 5, Refer:

https://brainly.com/question/19638051

#SPJ13

question 5 (10 points)

Answers

The answer is 924
You calculate the parenthesis
Then the exponents
After that u multiply and divide (left to right)
Add and subtract (left to right)

The rectangle below has an area of 70y^8+30y^670y
8
+30y
6
70, y, start superscript, 8, end superscript, plus, 30, y, start superscript, 6, end superscript.
The width of the rectangle is equal to the greatest common monomial factor of 70y^870y
8
70, y, start superscript, 8, end superscript and 30y^630y
6
30, y, start superscript, 6, end superscript.
What is the length and width of the rectangle?



\text{Width} =Width=start text, W, i, d, t, h, end text, equals
\text{Length} =Length=start text, L, e, n, g, t, h, end text, equals

Answers

The width of the rectangle with area 70y⁸+ 30y⁶ is; 10y⁶.

Then the length of the rectangle with area 70y⁸+ 30y⁶ is; (7y² + 3).

What are the dimensions of the rectangle?

The area of a rectangle is given by;

Area = Length× width

Given that;

The area of the rectangle = 70y⁸ + 30y⁶

Now, 70y⁸ can be broken down to; 7×5×2×y×y×y×y×y×y×y×y

And 30y⁶ can be broken down to; 5×3×2×y×y×y×y×y×y

The common factors of 70y⁸ and 30y⁶ are = 5,2,y,y,y,y,y,y

Thus, the greatest common factor of 70y⁸ and 30y⁶ is;

5×2×y×y×y×y×y×y = 10y⁶

Therefore our area 70y⁸+ 30y⁶ can be factorized to get;

A = 10y⁶(7y² + 3)

The greatest common monomial of 70y⁸+ 30y⁶ is 10y⁶ which is the width of the rectangle.

Read more about Rectangle Dimensions at; https://brainly.com/question/25292087

#SPJ1

How many twelfths are in 3 2/3?

Answers

To make a mixed number from an improper fraction, divide the numerator by the denominator then there exists eight twelfths in two-thirds.

How to convert mixed numbers to improper fraction?

Multiply the full number by the denominator to convert a mixed number to an improper fraction. Include it in the numerator. Add that total to the denominator from the beginning.

To make a mixed number from an improper fraction, divide the numerator by the denominator. After division, the mixed number is produced in such a way that the obtained quotient becomes the entire number, the remainder becomes the new numerator, and the denominator remains constant.

Let the value be [tex]$3 \frac{2}{3}$[/tex].

Convert mixed numbers to improper fraction:

[tex]$a \frac{b}{c}=\frac{a \cdot c+b}{c}$[/tex]

substitute the values in the above equation, we get

[tex]$&3 \frac{2}{3}=\frac{3 \cdot 3+2}{3}=\frac{11}{3} \\[/tex]

Therefore, there exists eight twelfths in two-thirds.

To learn more about improper fraction refer to:

brainly.com/question/387381

#SPJ13

Which segments are skewed

Answers

the following segments: ed, cd, ag, ab

Answer:

  AB, DC, ED, GA

Step-by-step explanation:

You want to identify the segments that are skew to edge FH of cuboid ABCDEFGH.

Skew lines

Two lines are skew when no plane exists that can contain them both. That is, skew lines are not parallel and do not intersect.

Cuboid ABCDEFGH has 12 edges. Of those, 3 are parallel to FH, and 4 intersect FH. The remaining edges are skew to FH:

  AB, DC, ED, GA . . . . . segments skew to FH

7.25 to the nearest tenth

Answers

Answer:

7.3

Step-by-step explanation:

Above 5- round up

below 5- round down

What is the slope of the line that passes through the points (0, -10)
and (-4,-11)?

Answers

Answer: - 4 /1 x -10

Step-by-step explanation:

the rise is -4

the run is -1

the y intesept is -10

At midnight, the temperature was -5°C.

By 9 am the next morning, the temperature had increased by 3°C .

Work out the temperature at 9 am the next morning.

Answers

Answer:  -2 degrees Celsius

Explanation:

Draw a vertical number line as shown below.

Plot -5 on the number line which is 5 units below 0.

Then move up 3 units to arrive at -2

This shows us that -5+3 = -2

Ryan collected 2,650 baseball cards, football cards, and hockey cards. he has 3 times as many baseball cards as football cards. he has 150 more hockey cards than football cards. how many baseball cards does he have? ryan has baseball cards.

Answers

The calculated answer after simplification in which Ryan has number of baseball cards is: 357 cards

How to simplify statement problems?

In mathematics, the statement based problems can be solved by forming the equations. And later, simplify those equation by equating to each other. This method is very common for the complicated problems to make the calculation easy.

According to the question, Ryan collected 2650 cards. In which, he has more baseball cards compared to football cards.

Now, let us assume, baseball cards be 'B'; football cards be 'F' and hockey cards be 'H'.

As per the question, the formed equation from the statement for the simplification is:

B + F + H = 2650

3B = F and H + 150 = F

Substituting above values in the formed equation to get the simplify answer,

B + 3B + 3B - 150 = 2650 ⇒ 7B = 2500 ⇒ B = 357

Now, the value of football cards is: F = 3(357) = 1071

And, the value of hockey cards is: H = 1071 - 150 = 921

Hence, after simplification Ryan has number of baseball cards is: 357 cards

To learn more about the simplification from the given link:

https://brainly.com/question/28780542

#SPJ13

a credit score is used by credit agencies​ (such as mortgage companies and​ banks) to assess the creditworthiness of individuals. values range from 300 to​ 850, with a credit score over 700 considered to be a quality credit risk. according to a​ survey, the mean credit score is . a credit analyst wondered whether​ high-income individuals​ (incomes in excess of​ $100,000 per​ year) had higher credit scores. he obtained a random sample of ​high-income individuals and found the sample mean credit score to be with a standard deviation of . conduct the appropriate test to determine if​ high-income individuals have higher credit scores at the level of significance.

Answers

There is insufficient evidence to conclude that those with the high incomes are the ones that have the more credit scores.

How to carry out the hypothesis test

We have the bar x = 705.9

The value of n = 34

mean = 724 . 1

standard deviation = 83.4

the level of significance = 5 percent level

The Hypothesis formulation

The null hypothesis H0 = u 705.9

alternative hypothesis = H1 u > 705.9

The t test is given as x - u / s /√n

= (724.1 - 705.9) / (83.4 / √34)

= 1.2725

The degree of freedom = 34 - 1 = 33

The test that we have here is a right tailed test

The p value for the test is given as probability of p (t33 > t)

= 0.1064

From the solution that we have here, we can see that the p value that has been calculated is greater than the level of significance.

Given that this is the case, the decision rule would be for us not to reject the null hypothesis.

There is insufficient evidence to the claim that the high income persons have the greater credit scores.

Read more on hypothesis testing here:

https://brainly.com/question/15980493

#SPJ1

Find the value of x in the triangle below:
X =
55°
Blank 1:
X


please hurry

Answers

Answer:

35

Step-by-step explanation:

90-55= 35 degress

I need the symetry so please be correct

Answers

Answer:

The Point of symmetry is (5, 6) and not (-5, -6).

Question 10 (1 point)
What are the coordinates of the midpoint of segment whose endpoints are A(-2,7) and B(4,-2)?
Make sure to use decimals, if necessary, and write with no spaces in the format (x,y) with you solving for x and y
A/

Answers

The midpoint of the line segment given is (1, 4.5)

Midpoint of a Line

In geometry, the midpoint is the middle point of a line segment. It is equidistant from both endpoints, and it is the centroid both of the segment and of the endpoints. It bisects the segment equally into two halves.

The formula of midpoint of a line is given as

[tex]M_x_y = \frac{x_1+x_2}{2}, \frac{y_1+y_2}{2}[/tex]

A = (-2, 7)B = (4, -2)

Substituting the given values;

[tex]M_x_y = \frac{x_1+x_2}{2}, \frac{y_1+y_2}{2}\\M_x_y = \frac{-2+4}{2}, \frac{7+2}{2} \\M_x_y = 1, \frac{9}{2} \\M_x_y = 1, 4.5[/tex]

The midpoint of the line AB is equal to (1, 4.5)

Learn more on midpoint of a line here;

https://brainly.com/question/5566419

#SPJ1

on Friday, 280 Fry Middle school students purchase pizza in the cafeteria.

Answers

What is your answer?

Determine the rate of change on the graphs below ​

Answers

Answer:

a. 3

b. -3/2

c. 4/3

d. -5/3

e. 0

f. undefined

Step-by-step explanation:

Convert into a mixed number.
19/01
7

Answers

Answer: 2 3/7

Step-by-step explanation:

Derek says the dilation was a reductionDelilah says the dilation was an enlargement who is correct and why

Answers

Delilah is correct; it is

Here, we want to check who was right

Mathematically, we can dilate a figure by reduction or enlargement by the use of a specific scale factor

When the scale factor is less than 1, it is a reduction

However, if the scale factor is greater than 1, it is an enlargement

The original point is (x,y)

From the image coordinates given, we see that the scale factor is 6/5 which is greater than 1

Thus, the dilation is an enlargement and Delilah was right

While sorting some change into piggy banks, Kyle put 4 coins in the first piggy bank, 6 coins in the second piggy bank, 9 coins in the third piggy bank, 13 coins in the fourth piggy bank, and 18 coins in the fifth piggy bank. If this pattern continues, how many coins will Kyle put in the sixth piggy bank?

Answers

Answer:24

Step-by-step explanation:4+2=6,6+3=9,9+4=13,13+5=18,18+6=24

Answer:

24

Step-by-step explanation:

because in first piggy bank has 4 coins and second has 6 . difference=6-4=2

second has 6 coins and third has 9 . difference= 9 - 6 = 3 third has 9 coins and fourth has 13 . difference=13-9=4

fourth has 13 and fifth has 18 coins difference is 18 -13 =5

then,sixth has 24 because all have 2 3 4 5 difference then six has 6

Bruno works for Poseidon Pool Service. He needs to drain a 38,000-gallon pool so that he can repair the cracked plaster. The Pool drains 12 gallons per minute.


Write an equation to represent the pool draining in slope-intercept form.


How long will it take for the pool to fully drain? Explain using numbers and words


Bruno needs to refill the pool now that the plaster is repaired. The Pool fills 16 gallons per minute.


Write an equation in slope intercept form to represent the pool filling back up.

*

How long will it take for the pool to fill back up? Explain using words and numbers..

*

Answers

The equation representing the pool draining in slope-intercept form is y = -12x + 38000 and the time for the pool to fully drain is 3167 minutes.

What is a linear equation?

It is described as the relationship between two variables, and a straight line results from plotting the graph of the linear equation.

The equation is referred to as a linear equation in one variable if just one variable is contained in the linear equation.

As we know,

The standard form of the straight line is:

y = mx + b

The slope in this case is m, and the y-intercept is b.

From the data given:

y = -12x + 38000

Plug y = 0 to find how long it will take for the pool to fully drain:

0 = -12x + 38000

x = 3167 minutes

To fill it back up, the y-intercept is 0.

The slope is given M = 16

y = 16x

Now plug y = 38000 to find how long it will take for the pool to fill back up

38000 = 16x

x = 2375 minutes

Thus, the equation representing the pool draining in slope-intercept form is y = -12x + 38000 and the time for the pool to fully drain is 3167 minutes.

Learn more about the linear equation here:

brainly.com/question/11897796

#SPJ1

What numbers are equivalent to 4/8?

Answers

Answer:numbers that are equal to 4/8 would be 2/4 and 1/2 and are you multiplying up?

Step-by-step explanation:

Use the figure shown. Let point D be at (-4,1) . Use the sides of triangle BDA to find the slope of the line.

Answers

The slope of the line shown in the figure having equation x + 2y = 0 is 1/2.

The point A is at ( -2 , 1 ), point B is at ( -4 , 2 ) and Point C is at ( -2 , 2 ).

Let point D be at ( -4 , 1 ).

Now the new triangle is ΔBDA and the corner points are A is at ( -2 , 1 ), point B is at ( -4 , 2 ) and  point D is at ( -4 , 1 ).

The equation of line passing through two points  is given by:

[tex](y-y_{1} ) = (\frac{y_{2}-y_{1} }{x_{2} - x_{1} })(x - x_{1} )[/tex]

The equation of line passing through two points A( -2 , 1 ) and B( -4 , 2 ) IS

y - 1 = ((2-1)/(-4-(-2)))  (x-(-2))

y - 1 = (1/(-2)) (x+2)

2y -2 = -x -2

x + 2y = 0

The equation of straight line is x + 2y = 0

The equation in slope intercept form can be written as,

y = x/2 + 0

Slope = m = 1/2

Intercept = c = 0

So, we can conclude that 1/2 is the slope of the line .

To know more about Slope - intercept form refer to link:

https://brainly.com/question/1884491

#SPJ1

The volume, V, of a container is directly proportional to the cube of its height, h. When its height
is 4 cm, its volume is 48 cm3.
1.What is the volume if the height is 3 cm?
2.If the height is doubled to 6 cm, how many times bigger will the volume be?

Answers

Answer:

see explanation

Step-by-step explanation:

given that V is directly proportional to h³ , then the equation relating them is

V = kh³ ← k is the constant of proportion

to find k use the condition V = 48 when h = 4 , then

48 = k × 4³ = 64k ( divide both sides by 64 )

[tex]\frac{48}{64}[/tex] = k , then k = 0.75

V = 0.75h³ ← equation of proportion

1

when h = 3 , then

V = 0.75 × 3³ = 0.75 × 27 = 20.25 cm³

2

when h = 6 , then

V = 0.75 × 6³ = 0.75 × 216 = 162 cm³

then

162 ÷ 20.25 = 8

the volume will be 8 times bigger

Evaluate the expression when a = -6.9 and b = -1.9,-66+2a

Answers

Evaluating Algebraic Expressions

We are given the expression:

-6b + 2a

And it's required to find the value of the expression for a=-6.9 and b=1.9

Substituting the values:

-6(-1.9) + 2(-6.9)

= 11.4 - 13.8

Operating:

=-2.4

The value is -2.4

The coordinates of the vertices of quadrilateral JKLM are J(-4,
1), K(2, 3), L(5,
3), and M(0,
5)
Drag and drop the choices into each box to correctly complete the sentences.




Answers

Answer:

Slope:

JK = (1/3)

LK = (-2)

ML = (2/5)

MJ = (-3/2)

Quadrilateral JKLM is not a parallelogram because the slopes don't match.

Step-by-step explanation:

Slope = m

JK = (-4, 1), (2, 3)

       (x₁, y₁)  (x₂, y₂)

        y₂ - y₁         3 - 1               2            2           1

m = ------------ = ------------- = ---------- = --------- = -------

        x₂ - x₁         2 - (-4)         2 + 4         6           3

----------------------------------------------------------------------------------------------------------

LK = (5, -3), (2, 3)

       (x₁, y₁)  (x₂, y₂)

        y₂ - y₁         3 - (-3)        3 + 3          6

m = ------------ = ------------- = ---------- = --------- = -2

        x₂ - x₁         2 - 5              -3           -3

----------------------------------------------------------------------------------------------------------

ML = (0, -5), (5, -3)

       (x₁, y₁)  (x₂, y₂)

        y₂ - y₁        -3 - (-5)      -3 + 5         2

m = ------------ = ------------- = ---------- = ---------

        x₂ - x₁         5 - 0              5            5

----------------------------------------------------------------------------------------------------------

MJ = (0, -5), (-4, 1)

       (x₁, y₁)  (x₂, y₂)

        y₂ - y₁        1 - (-5)         1 + 5          6          -3

m = ------------ = ------------- = ---------- = --------- = -------

        x₂ - x₁         -4 - 0            -4            -4          2

----------------------------------------------------------------------------------------------------------

I hope this helps!

If tomorrow is Friday, then today is Thursday.

Answers

That means yesterday was Wednesday

One employee of a computer store is paid a base salary of 2000 a month plus an 8% commision on all sales over 7000 during the month. how much must the employee sell in one month to earn a total of 4000 for the month

Answers

According to this, the answer is that the employee must sell $32,000 in one month to earn a total of $4,000 for the month.

From the information given, the equation would indicate that the total salary is equal to the base salary plus the result of subtracting $7,000 from the amount of sales for 8%:

2000+(x-7,000)*0.08=4,000, where x is the amount of sales

Now, you can solve for x:

2,000+0.08x-560=4,000

0.08x+1440=4,000

0.08x=4,000- 1440

x=2,560/0.08

x=32,000

According to this, the answer is that the employee must sell $32,000 in one month to earn a total of $4,000 for the month.

learn more about of sells here

https://brainly.com/question/17617596

#SPJ4

Please help me with this.

Answers

least common mutiple of...

6 and 8: 24

8 and 12: 24

12 and 9: 36

6 and 9: 18

Write an equation for the intervals of a parabola with x-intercepts at (3,0) and (9,0) that passes through the point (10,-7).

Help is always greatly appreciated.

Answers

[tex]\quad \huge \quad \quad \boxed{ \tt \:Answer }[/tex]

[tex]\qquad\displaystyle \tt \rightarrow \: y= - {x}^{2} +12x - 27[/tex]

____________________________________

[tex] \large \tt Solution \: : [/tex]

The values of x where a parabola cuts the x - axis (y = 0) are the roots of the quadratic equation.

I.e 3 and 9 for the given problem.

and the equation can be represented as :

[tex]\qquad\displaystyle \tt \rightarrow \: y = a(x - x1)(x- x2)[/tex]

where, x1 and x2 are the roots of the quadratic equation, a is a constant value (depicting strech in curve)

Now, plug in the values :

[tex]\qquad\displaystyle \tt \rightarrow \: y= a(x- 3)(x - 9)[/tex]

[tex]\qquad\displaystyle \tt \rightarrow \: y = a( {x}^{2} -9x - 3x +27)[/tex]

[tex]\qquad\displaystyle \tt \rightarrow \: y= a( {x}^{2} -12 +27)[/tex]

Now, we need to find the value of a, for that let's use the coordinates of a point lying on the curve (10 , -7)

[tex]\qquad\displaystyle \tt \rightarrow \: - 7= a( {10}^{2} -12(10) +27)[/tex]

[tex]\qquad\displaystyle \tt \rightarrow \: - 7= a(100- 120+27)[/tex]

[tex]\qquad\displaystyle \tt \rightarrow \: - 7= a( 7)[/tex]

[tex]\qquad\displaystyle \tt \rightarrow \: a = ( - 7) \div ( 7)[/tex]

[tex]\qquad\displaystyle \tt \rightarrow \: a = -1[/tex]

Now, we got all required values. let's plug the value of a in equation, and we will get the required equation of parabola.

[tex]\qquad\displaystyle \tt \rightarrow \: y= -( {x}^{2} -12x +27)[/tex]

[tex]\qquad\displaystyle \tt \rightarrow \: y= - {x}^{2} +12x - 27[/tex]

Answered by : ❝ AǫᴜᴀWɪᴢ ❞

Answer:

[tex]y=-x^2+12x-27[/tex]

Step-by-step explanation:

Intercept form of a quadratic equation

[tex]\boxed{y=a(x-p)(x-q)}[/tex]

where:

p and q are the x-intercepts.a is some constant.

Given x-intercepts:

(3, 0)(9, 0)

Substitute the given x-intercepts into the formula:

[tex]\implies y=a(x-3)(x-9)[/tex]

To find a, substitute the given point (10, -7) into the equation and solve for a:

[tex]\implies a(10-3)(10-9)=-7[/tex]

[tex]\implies a(7)(1)=-7[/tex]

[tex]\implies 7a=-7[/tex]

[tex]\implies a=-1[/tex]

Therefore, the equation of the function in factored form is:

[tex]\implies y=-(x-3)(x-9)[/tex]

Expand the brackets:

[tex]\implies y=-(x^2-9x-3x+27)[/tex]

[tex]\implies y=-(x^2-12x+27)[/tex]

[tex]\implies y=-x^2+12x-27[/tex]

Therefore, the equation of the function in standard form is:

[tex]\boxed{y=-x^2+12x-27}[/tex]

Arthur crafts miniature chocolate dollhouses which he sells for $24 each. arthur has calculated the breakeven level of revenues for his business at $1,720 of sales. the dollhouses have a variable cost of $12 to produce per unit. what are arthur's fixed costs?

Answers

Answer: you mom loves your neighbor

Step-by-step explanation: because the do things together

Other Questions
Draw the graph of your= 2x +1 PLS HELP ASAP!!!!!What is internal growth? (Not the business kind) $m\angle U=2x\degree$ , and $m\angle V=2\left(m\angle U\right)$ . Find the value of $x$ that makes $\angle U$ and $\angle V$ complementary angles A ball is thrown upward from the top of a building that is 160 feet tall. The height, H, of the ball T seconds after being thrown is given by the equation H = -16t^2 + 48t + 160. After how many seconds will the ball hit the ground?a. 2 b. 3c. 5d. 10 Expand the brackets and simplify theexpression below.8(3y + 2) + 4y 13. Nick is five years older than Will. The sum of their ages is 21. How old is Nick? Solve algebraically. The effect of COVID 19 are felt in the households as many loses their jobs, businesses have decline of incomes and there are a lot of restrictions in movements which cause the consumption of goods and services to reduced. Along this premise, how can the financial system help the households and businesses in this time of pandemic? what letter name is 5th above E? is the number 19 a prime or composite number Compute the probability of randomly drawing 10 cards from a deck and getting exactly 5 spades.Enter the exact answer. Determine whether the relation defines a function, and give the domain and range.{(-8, 7),(4.7).(-9, 1); 1) speed of bob?2)Show calculation for the speed of the bob:(write formula first, then insert value with unit, then find the result with unit) when should a firm consider expanding from strictly domestic trade to international trade to international trade? when should it consider becoming further involved in international trade? what factors might affect this firm's decisions in each case? Complete the following statement.a0 Todos los das yo _________ clases de yoga. A. tenemos B. tiene C. tengo D. tienen (8r 2 7r9)+(r 2 +r) According to the online content article, what is the main difference between a culture and a civilization?A.There is no difference between the two terms.B.Civilization refers to a groups way of life, while culture only looks at particular elements.C.Culture refers to a groups way of life, while civilization refers to an advanced or complex group that coexists near other cultural groups.D.Culture refers to the different elements that make a cultural group unique, while civilization refers to the similarities found amongst all the groups when compared to each other.Please select the best answer from the choices providedABCD How do I solve with fractions? (with answers) 132 days to 125.4 days percent of change Choose any topic and then write a "philosophy" on that topic in the same vein that Poe did with furniture. plss help i will give brainlest